PG Interaction

High School/JEE/NEET/IPhO Physics | 17-19 Yrs

Please select any topic shown in the side navigation bar to participate.

Post your question

 
  1.  
  2. undefined
    7 months ago by Nitish Kumar Yadav

    Post your answer here

     
  3.  
  4. Sir why B{i} - B{f}?
    2 years ago by Dibyojyoti Bhattacharjee

    Post your answer here

     
  5.  
  6. Sir, what will happen if we consider Phi_F to be -Bpia^2and the Phi_i to be Bpia^a??
    2 years ago by Dibyojyoti Bhattacharjee

    Post your answer here

     
  7.  
  8. sir in the 3rd case of this video that is the bulb case if we introduce a variable mag field then the needle frequently shows defelctions towards the right and left..So,my question is will it perform SHM,and if it does what are the parametres that can be calculated and can we calculate the time period of that..If yes then HOW???
    2 years ago by Mani Raja

    Post your answer here

     
  9.  
  10. Que. A copper disc rotates about an axis which is perpendicular to the disc. Now of magnetic field exist in the region and is present perpendicular to the disc. A potential difference is said to exist between the axis and rim of the disc i.e. an emf is induced. Sir my question is that by faradays law we know that emf is induced only if total magnetic flux linked with the conductor changes but in this case it doesn't appears to change as neither the magnetic field nor the area of the disc in contact of the field is changing so why emf is induced ?
    3 years ago by Awnish Shahi

    Post your answer here

     
  11.  
  12. Sir,in solved example 6 u told that EMF induced is 0 through circular part, but solved example 12 you are saying EMF is induced in circular part also, Please clarify it sir.
    3 years ago by P.Anurag

    Post your answer here

     
  13.  
  14. Sir, I understood the creation of pole at B and - pole at A. But could you please explain how are we finding that A is low Potential and B is high potential!!
    4 years ago by

    Post your answer here

     
  15.  
  16. Respected Sir, are we able to directly multiply N to BA to get phi=BAN as magnetic flux is a scalar??
    4 years ago by

    Post your answer here

     
  17.  
  18. Sir, what is the meaning of magnetic lines are cut??
    4 years ago by

    Post your answer here

     
  19.  
  20. can you please about decay currents?
    5 years ago by Veda Kumar

    Post your answer here

     
  21.  
  22. why the normal reaction of the rails is not taken in consideration???
    5 years ago by Shark Roy

    Post your answer here

     
  23.  
  24. sir if we apply Lenz law then a cloclwise current flows in the circuit to compensate the decrease in magnetic flux.In that case D should be at higher potential than A as current flows from higher to lower potential.Then why palm rule is showing A at higher potential??
    5 years ago by Shark Roy

    Post your answer here

     
  25.  
  26. why cant we use h=?eR.dt ?
    5 years ago by Jazib Dawre

    Post your answer here

     
  27.  
  28. sir electrons flow from higher to lower potential that b is low potential and a is at high potential
    5 years ago by vishal akula

    Post your answer here

     
  29.  
  30. Sir in your previous video you said that a clockwise current produces an inward magnetic field then how in this case magnetic field develops in outward direction due to the current induced in wire AB ?
    6 years ago by Dharamjeet Singh
    Ans 1 ->
    Use RHTR to find the direction . Or better watch sir previous videos .
    6 years ago by Dharamjeet Singh

    Post your answer here

     
  31.  
  32. the integration B dot with S which is equal to the flux is the line integral or surface integral
    6 years ago by Nilesh Singh
    Ans 1 ->
    surface integral
    5 years ago by
    Ans 2 ->
    surface integral
    2 years ago by Ankalugari Gangadhar

    Post your answer here

     
  33.  
  34. sir in this is b inward or outward\
    6 years ago by yo honey singh

    Post your answer here

     
  35.  
  36. How to take time
    6 years ago by Madem devi

    Post your answer here

     
  37.  
  38. Sir formula for induced emf in ring
    6 years ago by Madem devi

    Post your answer here

     
  39.  
  40. sir , My doubt is regarding video 15 of electromagnetic induction. When you said thetha=pi ,B will come out of the region and A will enter it and the current will be clockwise. But if we consider that rotation ,whole semicircular wire will be inside of the region with A and B on the boundary and so more flux will come out of it and hence current will be in anticlockwise direction to oppose it, But in the video you have taken it clockwise. Correct me if i am wrong!
    6 years ago by Michael jackson

    Post your answer here

     
  41.  
  42. How to approach the question if the axis of rotation lies on the rod? For example: Find the induced emf in a rod rotating in uniform magnetic field about its midpoint. Please reply ASAP.
    6 years ago by NAMAN MISHRA

    Post your answer here

     
  43.  
  44. Sir, why electric field is produced if wire is a neutral body which cannot produce electric field and can produce only magnetic field
    6 years ago by Pratyush Ranjan Roul

    Post your answer here

     
  45.  
  46. What is the potential difference between the two points in an induced electric field.please explain
    7 years ago by Gandham.yesu nageswararao

    Post your answer here

     
  47.  
  48. sir how to know whether we have to use fleming right hand rule or left hand rule while solving numericals on emi.
    7 years ago by sneha sahoo

    Post your answer here

     
  49.  
  50. sir does brightness of bulb remains same in the whole process
    7 years ago by ariya
    Ans 1 ->
    No, it depends upon the rate of flux variation which affects the emf induced...
    7 years ago by Physics Galaxy

    Post your answer here

     
  51.  
  52. Sir, why emf induced in a conducting ring is zero?
    7 years ago by Aár Káy Singh
    Ans 1 ->
    If ring is at rest in static magnetic field or moving in such a way that its area is normal to magnetic field direction then flux through ring remain constant hence no emf is induced in it as no variation in flux... but if ring starts rotating or change its orientation then flux through the ring will change and emf will be induced in it...
    7 years ago by Physics Galaxy

    Post your answer here

     
  53.  
  54. Sir,You have made video lectures on LC , LR and LCR circuits for an AC Source .... Can you PLEASE make the same for dc source as well... Thanking you in advance..
    7 years ago by Kaustav Ghosh
    Ans 1 ->
    For LR, LC and RC for DC Sources these are already there in topics of Self Induciton, LC Oscillations and Current electricity... you can watch these...
    7 years ago by Physics Galaxy

    Post your answer here

     
  55.  
  56. a thin semicicular conducting ing (PQR) of radius r if falling with its plane vetical in a horizontal magnetic field B. the potential difference developed across the ring when its speed is v, is
    7 years ago by Alekhya
    Ans 1 ->
    If ring is moving in such a way that it is cutting magnetic flux then across two points of the ring a potential difference will be induced but loop emf will be zero if total flux remain constant so no induced current will flow in it...
    7 years ago by Physics Galaxy

    Post your answer here

     
  57.  
  58. Sir while deriving the equation for emf induced we took electric field in the rod to be constant??H ow can we say that??
    8 years ago by jeevan99
    Ans 1 ->
    As during motion, all electrons are experiencing same magnetic force along the length of the rod, in steady state electric force balances this magnetic force hence the electric force on all electrons must be same... however here we are assuming all electrons come to rest but they have some random motion due to thermal agitation which we are ignoring here...
    8 years ago by Physics Galaxy

    Post your answer here

     
  59.  
  60. sir please clear my doubt. if the loop is drawn out according to Lenz's law it ha to be opposed so the current in the loop should produce a magnetic field which is opposite to the original magnetic field. the current induced must be anti-clockwise?
    8 years ago by job
    Ans 1 ->
    When loop is drawn out flux through the loop decreases and to oppose the change in magnetic field (acc or Lenz's Law) it should increase the field in same direction thats why induced current is clockwise...
    8 years ago by Physics Galaxy

    Post your answer here

     
  61.  
  62. sir can you please explain the functioning of moving coil galvanometer that why we find the torque of only the mag field parallel to loop only?
    8 years ago by Parth gupta

    Post your answer here

     
  63.  
  64. Sir please tell me how can we understand why does di/dt decrease with time and how can we analyse the r-l series circuit graphs on basis of pure physics
    8 years ago by Harshit Rawat

    Post your answer here

     
  65.  
  66. sir please provide solutions to pg brainstormers
    8 years ago by Deep Surve

    Post your answer here

     
  67.  
  68. Sir please provide the solution to level 1 set 3 question 2.
    8 years ago by Ebin Sunny Varghese

    Post your answer here

     
  69.  
  70. Sir, given that in the metal lattice of a magnet the atoms / electrons are aligned vertically and in sync to create and support the magnetic field created, can we not suppose that these atoms / electrons ( domains ) are in sync left and right of each other. In other words ALL motions of these magnetic field producing electrons are completely in sync through out the structure.. Now, given that this is true, shouldn't a disc magnet in rotation on its axis produce electrons that are blue shift during part of the orbit, and red shift during the other part, doesn't create a relativistic magnetic field ?
    9 years ago by eric reed

    Post your answer here

     
  71.  
  72. Sir, a disc magnet ( north on top ) is given a negative charge, and the disc magnet is rotated clockwise.... Will this negative charge create its own magnetic and support the existing mag field ? Also we can reverse rotation and it would seem the negative charge would diminish the existing field... how are we to look at this ?
    9 years ago by eric reed
    Ans 1 ->
    Yes you are right... depending upon direction of rotation the magnetic field due to charge on disc may support or oppose its own magnetic field...
    9 years ago by Physics Galaxy

    Post your answer here

     
  73.  
  74. Sir, why in case of DC motor , if current thr armature is minimum, then emf induced is maximum ?
    9 years ago by Anand Rahangdale
    Ans 1 ->
    In case of DC Motor when emf induced is maximum, it opposes the external supply voltage so the overall potential difference across armature is minimum that why current is minimum...
    9 years ago by Physics Galaxy

    Post your answer here

     
  75.  
  76. sir can you please explain that why emf is induced in a closed loop as according to kirchoff's law potential drop across a closed loop is zero
    9 years ago by shivam sharma
    Ans 1 ->
    Due to time variation of magnetic fields, non conservative electric field is developed/induced which induces the emf whereas in kirchoff's laws we use conservative electric field established due to a battery/potential source in a circuit...
    9 years ago by Physics Galaxy

    Post your answer here

     
  77.  
  78. sir can you tell me the derivation for the magnetic field generated by a bar magnet on axial line
    9 years ago by arindam thapliyal
    Ans 1 ->
    It is here - http://physicsgalaxy.com/lectures/1/64/1481/Magnetic%20Induction%20due%20to%20a%20Bar%20Magnet%20on%20its%20Axis#4
    9 years ago by Physics Galaxy

    Post your answer here

     
  79.  
  80. Sir can you please explain why when a bar magnet is brought near a coil , current is induced in it?
    9 years ago by Vishwajeet
    Ans 1 ->
    due to the movements of charges i.e electron due to the megnetic force
    9 years ago by rajat
    Ans 2 ->
    This is due to change in flux in the closed loop !
    9 years ago by Anand Rahangdale

    Post your answer here

     
  81.  
  82. Dear Sir could you please explain why the length is considered from point A to B in a random shape conductor? I know that length should be perp to velocity.
    10 years ago by karan
    Ans 1 ->
    because we consider vector L. vectorially if you the effective length will turn out to be AB.
    9 years ago by shivani shende

    Post your answer here

     
  83.  
  84. Sir please explain Why end A is at higher potential? O should be the positive terminal of cell formed!
    10 years ago by Akhil Mehta

    Post your answer here

     
  85.  
  86. sir how we will be able to know that it is time varying magnetic field like there was iit this year paragraph
    10 years ago by venu sharma

    Post your answer here